LSAT and Law School Admissions Forum

Get expert LSAT preparation and law school admissions advice from PowerScore Test Preparation.

User avatar
 Dana D
PowerScore Staff
  • PowerScore Staff
  • Posts: 117
  • Joined: Feb 06, 2024
|
#105752
Complete Question Explanation

Assumption. The correct answer choice is (C).

The stimulus makes the argument that incumbent political leaders will remain in power after the upcoming elections because voter confidence in the economy favors incumbents. However, the only premise to support this idea is that investors are confident in the economy. Thus, this argument only makes sense if we can link the ideas of investors attitudes and voter attitudes - we're looking for an answer choice that does this.

Answer choice (A): We don't need this to be true. We don't need to make a case for why voters might want to retain incumbent leaders, we need to use the journalist's own argument to assess what assumption they are relying on.

Answer choice (B): Don't need this to be true - we still need to link investors and voters.

Answer choice (C): This is the correct answer choice.
This answer equates voter choice with investor choice. Adding this information into the stimulus would let us say investors are confident of increased growth of the economy :arrow: their economic attitudes don't differ much from voters in general :arrow: voters in general are confident in the economy :arrow: voter confidence in the economy factors incumbent political leaders :arrow: party in power is likely to retain power

Answer choice (D): If this is true, why would the party in power stay in power? The recent state of political news has been depressing, so wouldn't voters want a new party in power? This has to be false.

Answer choice (E): If this is true, how can we say whether the party in power will stay in power? We don't know how much they won by, and if it was very close, maybe some voters don't stay loyal and the incumbent party is ousted. Also, this answer doesn't provide the missing link of information we're looking for which ties investors and voters together.
 thedramallama
  • Posts: 2
  • Joined: Oct 23, 2019
|
#71506
What's the difference between AC B and C? Is B incorrect because we can't assume that investors are the same as voters?
 Jeremy Press
PowerScore Staff
  • PowerScore Staff
  • Posts: 1000
  • Joined: Jun 12, 2017
|
#71525
Hi dramallama,

Yes, that's exactly right. We don't know in the stimulus that the investors (whose investment choices are being taken as indicative of voters' economic confidence) are actually voters. Thus, answer choice B isn't necessary, and might not even be helpful to the argument, if the investors in the premises turned out not to be voters. Answer choice C makes the more general connection necessary to tie the premises to the conclusion: there must be at the very least a similarity between the confidence investors are showing in the economy, and the confidence that the voting population has in the economy.

I hope this helps!

Jeremy
 theamazingrace
  • Posts: 59
  • Joined: Oct 17, 2020
|
#81013
Is it the mention of polices in answer choices A and D that make them wrong?

Thanks
 Rachael Wilkenfeld
PowerScore Staff
  • PowerScore Staff
  • Posts: 1358
  • Joined: Dec 15, 2011
|
#81067
Hi Grace,

For assumption questions, we are asked to determine what is required for the argument to follow. Answer choice (A) talks about factors that cause stock market growth. But we don't need to know anything about stock market growth to lead to the conclusion that the incoming party will win. That is based on voter confidence in the economy. That means answer choice (D), information on how voters attribute responsibility, is also not relevant to the conclusion on voter confidence.

Hope that helps!
 agurrola
  • Posts: 1
  • Joined: Jul 06, 2023
|
#102786
Can this question please be explained? I made note that the conclusion is, "it is likely that the party now in power will retain power after the upcoming elections".

Premises offered:
- investors are confident of growth in the economy
- voter confidence in the economy favors incumbents

I selected answer choice B.
 Adam Tyson
PowerScore Staff
  • PowerScore Staff
  • Posts: 5153
  • Joined: Apr 14, 2011
|
#102792
I'll give a brief breakdown, agurrola! The argument is structured as follows:

Premise: Investors are buying stocks despite bad political news

Sub-Conclusion: Investors are confident the economy will grow

Premise: Voter confidence in the economy favors incumbents

Main Conclusion: Incumbents will probably win reelection

Notice the shift between the sub-conclusion, which is about just those investors who were mentioned in the first premise, to voters, who may or may not be the same people as the investors? That's a flaw in the argument. Perhaps the investors don't vote? Even if they do, maybe they are not a representative sample of voters generally?

The question asks us to identify a necessary assumption of the argument, and in this case a good approach might be to think of how the author would respond in order to fix that flaw. We call that the "Defender" approach to Assumption questions. The author has to take a position that defends the argument against attack, because if they do not, then their argument will be destroyed.

With this in mind, we should be looking for an answer that indicates that those investors ARE a representative sample of voters. Their confidence needs to be shared by most voters. Answer C does that. Answer B fails, because while it tells us something about how the investors are likely to vote, it tells us nothing about the broader group of ALL voters. So what if some rich folks are going to vote for the incumbents? How are ordinary people going to vote?

Look for gaps like that in arguments, and use them to prephrase answers to Assumption, Strengthen, Justify the Conclusion, Flaw, and Weaken questions.

Get the most out of your LSAT Prep Plus subscription.

Analyze and track your performance with our Testing and Analytics Package.